Double integral with logarithms [on hold]
$begingroup$
Faster method to calculate the exact solution of the following integral (based on the ideas of Fedor Petrov: https://mathoverflow.net/users/4312/fedor-petrov, Double integral with logarithms, URL (version: 2019-04-15): https://mathoverflow.net/q/328126):
$$Jequiv int_{0}^{1}{int_{0}^{1}{frac{ln x-ln y}{x-y}}}dxdy .$$
Since
$$fleft( x,y right)=frac{ln x-ln y}{x-y}=fleft( y,x right),$$
the surface $fleft( x,y right) $ is symmetric with respect to the bisector plane $x = y$; so,
$$frac{J}{2}=int_{0}^{1}{dxint_{0}^{x}{frac{ln x-ln y}{x-y}}}dy.$$
With the change of variable$$yequiv tx, tin left( 0, 1 right),$$
the integral
$$int_{0}^{x}{frac{ln x-ln y}{x-y}}dy,$$
is transformed into the following one that does not depend on $x$,
$$ Iequiv -int_{0}^{1}{frac{ln t}{1-t},}dt.$$
The integration on the unit square $(0, 0), (1, 0), (1, 1), (0,1)$ is reduced to the integration on the triangle $(0, 0), (1, 0), (1, 1).$
To solve the integral $I$ we will carry out the new change of variable,
$$sequiv 1-t,$$
by which $I$ is transformed into the integral that defines the dilogarithm, whose value for $s = 1$ coincides with the Riemann zeta $zeta left( 2 right)$,whose value is well known:
$$I=int_{1}^{0}{frac{ln left( 1-s right)}{s},}ds=text{L}{{text{i}}_{2}}left( 1 right)=zeta left( 2 right)=frac{{{pi }^{2}}}{6}.$$
Therefore, the solution to the proposed integral is
$$J=frac{{{pi }^{2}}}{3}.$$
Note. I've tried it by polylogarithmic transformations, but I couldn't get the result $frac{{{pi }^{2}}}{3}.$
integration
New contributor
$endgroup$
put on hold as off-topic by YCor, user44191, Pace Nielsen, Gerald Edgar, Piotr Hajlasz yesterday
This question appears to be off-topic. The users who voted to close gave this specific reason:
- "MathOverflow is for mathematicians to ask each other questions about their research. See Math.StackExchange to ask general questions in mathematics." – Pace Nielsen, Gerald Edgar, Piotr Hajlasz
If this question can be reworded to fit the rules in the help center, please edit the question.
|
show 6 more comments
$begingroup$
Faster method to calculate the exact solution of the following integral (based on the ideas of Fedor Petrov: https://mathoverflow.net/users/4312/fedor-petrov, Double integral with logarithms, URL (version: 2019-04-15): https://mathoverflow.net/q/328126):
$$Jequiv int_{0}^{1}{int_{0}^{1}{frac{ln x-ln y}{x-y}}}dxdy .$$
Since
$$fleft( x,y right)=frac{ln x-ln y}{x-y}=fleft( y,x right),$$
the surface $fleft( x,y right) $ is symmetric with respect to the bisector plane $x = y$; so,
$$frac{J}{2}=int_{0}^{1}{dxint_{0}^{x}{frac{ln x-ln y}{x-y}}}dy.$$
With the change of variable$$yequiv tx, tin left( 0, 1 right),$$
the integral
$$int_{0}^{x}{frac{ln x-ln y}{x-y}}dy,$$
is transformed into the following one that does not depend on $x$,
$$ Iequiv -int_{0}^{1}{frac{ln t}{1-t},}dt.$$
The integration on the unit square $(0, 0), (1, 0), (1, 1), (0,1)$ is reduced to the integration on the triangle $(0, 0), (1, 0), (1, 1).$
To solve the integral $I$ we will carry out the new change of variable,
$$sequiv 1-t,$$
by which $I$ is transformed into the integral that defines the dilogarithm, whose value for $s = 1$ coincides with the Riemann zeta $zeta left( 2 right)$,whose value is well known:
$$I=int_{1}^{0}{frac{ln left( 1-s right)}{s},}ds=text{L}{{text{i}}_{2}}left( 1 right)=zeta left( 2 right)=frac{{{pi }^{2}}}{6}.$$
Therefore, the solution to the proposed integral is
$$J=frac{{{pi }^{2}}}{3}.$$
Note. I've tried it by polylogarithmic transformations, but I couldn't get the result $frac{{{pi }^{2}}}{3}.$
integration
New contributor
$endgroup$
put on hold as off-topic by YCor, user44191, Pace Nielsen, Gerald Edgar, Piotr Hajlasz yesterday
This question appears to be off-topic. The users who voted to close gave this specific reason:
- "MathOverflow is for mathematicians to ask each other questions about their research. See Math.StackExchange to ask general questions in mathematics." – Pace Nielsen, Gerald Edgar, Piotr Hajlasz
If this question can be reworded to fit the rules in the help center, please edit the question.
5
$begingroup$
MSE is a right place for such type questions. Both Maple and Mathematica confirm $ frac{pi ^2}{3}$.
$endgroup$
– user64494
yesterday
9
$begingroup$
I do not think this is such a bad question. I would not expect even every research mathematician to come up with the answer right away. So I vote not to close.
$endgroup$
– RP_
yesterday
6
$begingroup$
@user64494 what is a general strategy: when you have an integral which you do not know how to calculate, how should you decide between MSE and MO?
$endgroup$
– Fedor Petrov
yesterday
6
$begingroup$
@user64494 being the author of several papers devoted to exact closed-form integration, I feel so old now.
$endgroup$
– Fedor Petrov
yesterday
2
$begingroup$
Learning techniques for a problem here could be illuminating (depending on ...) for future purposes of integration or summation that they relate to or approximated by (if you are into numerical values). Plus, cute arguments are aesthetically pleasing ...
$endgroup$
– T. Amdeberhan
yesterday
|
show 6 more comments
$begingroup$
Faster method to calculate the exact solution of the following integral (based on the ideas of Fedor Petrov: https://mathoverflow.net/users/4312/fedor-petrov, Double integral with logarithms, URL (version: 2019-04-15): https://mathoverflow.net/q/328126):
$$Jequiv int_{0}^{1}{int_{0}^{1}{frac{ln x-ln y}{x-y}}}dxdy .$$
Since
$$fleft( x,y right)=frac{ln x-ln y}{x-y}=fleft( y,x right),$$
the surface $fleft( x,y right) $ is symmetric with respect to the bisector plane $x = y$; so,
$$frac{J}{2}=int_{0}^{1}{dxint_{0}^{x}{frac{ln x-ln y}{x-y}}}dy.$$
With the change of variable$$yequiv tx, tin left( 0, 1 right),$$
the integral
$$int_{0}^{x}{frac{ln x-ln y}{x-y}}dy,$$
is transformed into the following one that does not depend on $x$,
$$ Iequiv -int_{0}^{1}{frac{ln t}{1-t},}dt.$$
The integration on the unit square $(0, 0), (1, 0), (1, 1), (0,1)$ is reduced to the integration on the triangle $(0, 0), (1, 0), (1, 1).$
To solve the integral $I$ we will carry out the new change of variable,
$$sequiv 1-t,$$
by which $I$ is transformed into the integral that defines the dilogarithm, whose value for $s = 1$ coincides with the Riemann zeta $zeta left( 2 right)$,whose value is well known:
$$I=int_{1}^{0}{frac{ln left( 1-s right)}{s},}ds=text{L}{{text{i}}_{2}}left( 1 right)=zeta left( 2 right)=frac{{{pi }^{2}}}{6}.$$
Therefore, the solution to the proposed integral is
$$J=frac{{{pi }^{2}}}{3}.$$
Note. I've tried it by polylogarithmic transformations, but I couldn't get the result $frac{{{pi }^{2}}}{3}.$
integration
New contributor
$endgroup$
Faster method to calculate the exact solution of the following integral (based on the ideas of Fedor Petrov: https://mathoverflow.net/users/4312/fedor-petrov, Double integral with logarithms, URL (version: 2019-04-15): https://mathoverflow.net/q/328126):
$$Jequiv int_{0}^{1}{int_{0}^{1}{frac{ln x-ln y}{x-y}}}dxdy .$$
Since
$$fleft( x,y right)=frac{ln x-ln y}{x-y}=fleft( y,x right),$$
the surface $fleft( x,y right) $ is symmetric with respect to the bisector plane $x = y$; so,
$$frac{J}{2}=int_{0}^{1}{dxint_{0}^{x}{frac{ln x-ln y}{x-y}}}dy.$$
With the change of variable$$yequiv tx, tin left( 0, 1 right),$$
the integral
$$int_{0}^{x}{frac{ln x-ln y}{x-y}}dy,$$
is transformed into the following one that does not depend on $x$,
$$ Iequiv -int_{0}^{1}{frac{ln t}{1-t},}dt.$$
The integration on the unit square $(0, 0), (1, 0), (1, 1), (0,1)$ is reduced to the integration on the triangle $(0, 0), (1, 0), (1, 1).$
To solve the integral $I$ we will carry out the new change of variable,
$$sequiv 1-t,$$
by which $I$ is transformed into the integral that defines the dilogarithm, whose value for $s = 1$ coincides with the Riemann zeta $zeta left( 2 right)$,whose value is well known:
$$I=int_{1}^{0}{frac{ln left( 1-s right)}{s},}ds=text{L}{{text{i}}_{2}}left( 1 right)=zeta left( 2 right)=frac{{{pi }^{2}}}{6}.$$
Therefore, the solution to the proposed integral is
$$J=frac{{{pi }^{2}}}{3}.$$
Note. I've tried it by polylogarithmic transformations, but I couldn't get the result $frac{{{pi }^{2}}}{3}.$
integration
integration
New contributor
New contributor
edited 9 hours ago
Jesús Álvarez Lobo
New contributor
asked yesterday
Jesús Álvarez LoboJesús Álvarez Lobo
353
353
New contributor
New contributor
put on hold as off-topic by YCor, user44191, Pace Nielsen, Gerald Edgar, Piotr Hajlasz yesterday
This question appears to be off-topic. The users who voted to close gave this specific reason:
- "MathOverflow is for mathematicians to ask each other questions about their research. See Math.StackExchange to ask general questions in mathematics." – Pace Nielsen, Gerald Edgar, Piotr Hajlasz
If this question can be reworded to fit the rules in the help center, please edit the question.
put on hold as off-topic by YCor, user44191, Pace Nielsen, Gerald Edgar, Piotr Hajlasz yesterday
This question appears to be off-topic. The users who voted to close gave this specific reason:
- "MathOverflow is for mathematicians to ask each other questions about their research. See Math.StackExchange to ask general questions in mathematics." – Pace Nielsen, Gerald Edgar, Piotr Hajlasz
If this question can be reworded to fit the rules in the help center, please edit the question.
5
$begingroup$
MSE is a right place for such type questions. Both Maple and Mathematica confirm $ frac{pi ^2}{3}$.
$endgroup$
– user64494
yesterday
9
$begingroup$
I do not think this is such a bad question. I would not expect even every research mathematician to come up with the answer right away. So I vote not to close.
$endgroup$
– RP_
yesterday
6
$begingroup$
@user64494 what is a general strategy: when you have an integral which you do not know how to calculate, how should you decide between MSE and MO?
$endgroup$
– Fedor Petrov
yesterday
6
$begingroup$
@user64494 being the author of several papers devoted to exact closed-form integration, I feel so old now.
$endgroup$
– Fedor Petrov
yesterday
2
$begingroup$
Learning techniques for a problem here could be illuminating (depending on ...) for future purposes of integration or summation that they relate to or approximated by (if you are into numerical values). Plus, cute arguments are aesthetically pleasing ...
$endgroup$
– T. Amdeberhan
yesterday
|
show 6 more comments
5
$begingroup$
MSE is a right place for such type questions. Both Maple and Mathematica confirm $ frac{pi ^2}{3}$.
$endgroup$
– user64494
yesterday
9
$begingroup$
I do not think this is such a bad question. I would not expect even every research mathematician to come up with the answer right away. So I vote not to close.
$endgroup$
– RP_
yesterday
6
$begingroup$
@user64494 what is a general strategy: when you have an integral which you do not know how to calculate, how should you decide between MSE and MO?
$endgroup$
– Fedor Petrov
yesterday
6
$begingroup$
@user64494 being the author of several papers devoted to exact closed-form integration, I feel so old now.
$endgroup$
– Fedor Petrov
yesterday
2
$begingroup$
Learning techniques for a problem here could be illuminating (depending on ...) for future purposes of integration or summation that they relate to or approximated by (if you are into numerical values). Plus, cute arguments are aesthetically pleasing ...
$endgroup$
– T. Amdeberhan
yesterday
5
5
$begingroup$
MSE is a right place for such type questions. Both Maple and Mathematica confirm $ frac{pi ^2}{3}$.
$endgroup$
– user64494
yesterday
$begingroup$
MSE is a right place for such type questions. Both Maple and Mathematica confirm $ frac{pi ^2}{3}$.
$endgroup$
– user64494
yesterday
9
9
$begingroup$
I do not think this is such a bad question. I would not expect even every research mathematician to come up with the answer right away. So I vote not to close.
$endgroup$
– RP_
yesterday
$begingroup$
I do not think this is such a bad question. I would not expect even every research mathematician to come up with the answer right away. So I vote not to close.
$endgroup$
– RP_
yesterday
6
6
$begingroup$
@user64494 what is a general strategy: when you have an integral which you do not know how to calculate, how should you decide between MSE and MO?
$endgroup$
– Fedor Petrov
yesterday
$begingroup$
@user64494 what is a general strategy: when you have an integral which you do not know how to calculate, how should you decide between MSE and MO?
$endgroup$
– Fedor Petrov
yesterday
6
6
$begingroup$
@user64494 being the author of several papers devoted to exact closed-form integration, I feel so old now.
$endgroup$
– Fedor Petrov
yesterday
$begingroup$
@user64494 being the author of several papers devoted to exact closed-form integration, I feel so old now.
$endgroup$
– Fedor Petrov
yesterday
2
2
$begingroup$
Learning techniques for a problem here could be illuminating (depending on ...) for future purposes of integration or summation that they relate to or approximated by (if you are into numerical values). Plus, cute arguments are aesthetically pleasing ...
$endgroup$
– T. Amdeberhan
yesterday
$begingroup$
Learning techniques for a problem here could be illuminating (depending on ...) for future purposes of integration or summation that they relate to or approximated by (if you are into numerical values). Plus, cute arguments are aesthetically pleasing ...
$endgroup$
– T. Amdeberhan
yesterday
|
show 6 more comments
1 Answer
1
active
oldest
votes
$begingroup$
By symmetry we have $J/2=int_0^1 dx int_0^x f(x, y) dy$ where $f(x, y) $ is your integrand. Integrating against $y$ for fixed $x$ we denote $y=tx$, $t$ varies from 0 to 1 and the integral against $y$ reads as $-int_0^1 frac{log t} {1-t}dt$. It does not depend on $x$ and is well known to be equal to $pi^2/6$ (you may use the geometric progression expansion $frac{1}{1 - t} =sum_{n>0 } t^{n-1}$ and integrate term-wise to get $sum 1/n^2$).
$endgroup$
add a comment |
1 Answer
1
active
oldest
votes
1 Answer
1
active
oldest
votes
active
oldest
votes
active
oldest
votes
$begingroup$
By symmetry we have $J/2=int_0^1 dx int_0^x f(x, y) dy$ where $f(x, y) $ is your integrand. Integrating against $y$ for fixed $x$ we denote $y=tx$, $t$ varies from 0 to 1 and the integral against $y$ reads as $-int_0^1 frac{log t} {1-t}dt$. It does not depend on $x$ and is well known to be equal to $pi^2/6$ (you may use the geometric progression expansion $frac{1}{1 - t} =sum_{n>0 } t^{n-1}$ and integrate term-wise to get $sum 1/n^2$).
$endgroup$
add a comment |
$begingroup$
By symmetry we have $J/2=int_0^1 dx int_0^x f(x, y) dy$ where $f(x, y) $ is your integrand. Integrating against $y$ for fixed $x$ we denote $y=tx$, $t$ varies from 0 to 1 and the integral against $y$ reads as $-int_0^1 frac{log t} {1-t}dt$. It does not depend on $x$ and is well known to be equal to $pi^2/6$ (you may use the geometric progression expansion $frac{1}{1 - t} =sum_{n>0 } t^{n-1}$ and integrate term-wise to get $sum 1/n^2$).
$endgroup$
add a comment |
$begingroup$
By symmetry we have $J/2=int_0^1 dx int_0^x f(x, y) dy$ where $f(x, y) $ is your integrand. Integrating against $y$ for fixed $x$ we denote $y=tx$, $t$ varies from 0 to 1 and the integral against $y$ reads as $-int_0^1 frac{log t} {1-t}dt$. It does not depend on $x$ and is well known to be equal to $pi^2/6$ (you may use the geometric progression expansion $frac{1}{1 - t} =sum_{n>0 } t^{n-1}$ and integrate term-wise to get $sum 1/n^2$).
$endgroup$
By symmetry we have $J/2=int_0^1 dx int_0^x f(x, y) dy$ where $f(x, y) $ is your integrand. Integrating against $y$ for fixed $x$ we denote $y=tx$, $t$ varies from 0 to 1 and the integral against $y$ reads as $-int_0^1 frac{log t} {1-t}dt$. It does not depend on $x$ and is well known to be equal to $pi^2/6$ (you may use the geometric progression expansion $frac{1}{1 - t} =sum_{n>0 } t^{n-1}$ and integrate term-wise to get $sum 1/n^2$).
edited yesterday
T. Amdeberhan
18.4k230132
18.4k230132
answered yesterday
Fedor PetrovFedor Petrov
52.3k6122240
52.3k6122240
add a comment |
add a comment |
5
$begingroup$
MSE is a right place for such type questions. Both Maple and Mathematica confirm $ frac{pi ^2}{3}$.
$endgroup$
– user64494
yesterday
9
$begingroup$
I do not think this is such a bad question. I would not expect even every research mathematician to come up with the answer right away. So I vote not to close.
$endgroup$
– RP_
yesterday
6
$begingroup$
@user64494 what is a general strategy: when you have an integral which you do not know how to calculate, how should you decide between MSE and MO?
$endgroup$
– Fedor Petrov
yesterday
6
$begingroup$
@user64494 being the author of several papers devoted to exact closed-form integration, I feel so old now.
$endgroup$
– Fedor Petrov
yesterday
2
$begingroup$
Learning techniques for a problem here could be illuminating (depending on ...) for future purposes of integration or summation that they relate to or approximated by (if you are into numerical values). Plus, cute arguments are aesthetically pleasing ...
$endgroup$
– T. Amdeberhan
yesterday